Finding the signal x(t) when given properties of fourier series coefficients

Click For Summary
SUMMARY

This discussion centers on determining the continuous-time periodic signal x(t) with a period of 3, given specific properties of its Fourier series coefficients a_k. The properties include a_{k} = a_{k+2}, a_{k} = a_{-k}, and two integral conditions that define the behavior of x(t) over specified intervals. The solution involves expressing x(t) as a Fourier series, identifying it as an even function, and utilizing the Dirac delta function to represent its values in the defined intervals. The final form of x(t) is derived from these properties, emphasizing the importance of understanding Fourier series and signal periodicity.

PREREQUISITES
  • Fourier series representation of signals
  • Properties of periodic functions
  • Dirac delta function and its applications
  • Fundamental frequency calculation
NEXT STEPS
  • Study the properties of Fourier series coefficients in detail
  • Learn about the Dirac delta function and its role in signal processing
  • Explore the implications of even and odd functions in Fourier analysis
  • Investigate periodic signal reconstruction techniques
USEFUL FOR

Students and professionals in electrical engineering, signal processing, and applied mathematics who are working with Fourier series and periodic signals will benefit from this discussion.

Jncik
Messages
95
Reaction score
0

Homework Statement



Suppose we are given the following information about a continuous-time periodic signal with period 3 and Fourier coefficients a_{k}

1. a_{k} = a_{k+2}
2. a_{k} = a_{-k}
3. \int_{-0.5}^{0.5}x(t)dt = 1
4. \int_{0.5}^{1.5}x(t)dt = 2

Determine x(t)

Homework Equations



if x(t) is a periodic input signal it can be expressed as

x(t) = \sum_{k=-\infty}^{+\infty}a_{k}e^{jk\omega_{0} t}

which is called the Fourier series of x(t), and \omega_{0} is the fundamental frequency of x(t)

also a_{k} = \int_{-\infty}^{+\infty} x(t) e^{-j \omega_{0} k t} dt

The Attempt at a Solution



I've tried to solve this problem many times, when I was reading about it in the Fourier series chapter.. I thought if I moved onto the next chapter which was Fourier transform it would help me but it didn't, so here I am again..

what I first found is that \omega_{0} = \frac{2\pi}{3}

hence x(t) = \sum_{k=-\infty}^{+\infty}a_{k}e^{jk\frac{2\pi}{3} t}

now from the second property we can understand that x(t) is an even signal... hence

if

x(t) <-> ak
x(-t) <-> a-k
x(t) = x(-t) => ak = a-k

now from this we can simplify the summation

x(t) = \sum_{k=0}^{+\infty}a_{k}(e^{jk\frac{2\pi}{3} t}+e^{-jk\frac{2\pi}{3} t})=\sum_{k=0}^{+\infty}a_{k} 2cos(\frac{k 2\pi t}{3})

now from the first property since a_{k} = a_{k+2}

we see from the frequency shifting property that

x(t) = x(t) e^{\frac{-j 4\pi t}{3}}

good, we have 2 other properties

from the third property since \int_{-0.5}^{0.5}x(t)dt = 1 then x(t) must be the dirac delta function in this interval, hence x(t) = \delta (t) for -0.5&lt;=t&lt;=0.5

and from the fourth property, again it's a shifted dirac delta with amplitude changed hence

x(t) = 2 \delta (t-1) for 0.5 &lt;= t &lt;= 1.5

well I have these results which I hope to be correct, how can I use them in order to find the final x(t)?

I've solved many such exercises but they were a lot easier.. there was parseval involved, some other properties, for example it was stated that x(t) was real and even etc.. this is the only exercise that I have difficulty to find a solution and I need to learn how to solve it because our professor likes these kind of exercises

thanks in advance
 
Last edited:
Physics news on Phys.org
This is more of an algebra problem. As a_{k} = a_{k+2}
and a_{k} = a_{-k} , two constants determine the series. (3) & (4) are precisely the equations which make this a problem of two equations in two unknowns.
 
which are the equations?

I mean, from the first two, we have equations about a_{k} but these just tell us some properties about the frequency shifting and also about the symmetry of the signal..

about the 3 and 4 I'm not sure at all, I could only figure out what the x(t) would be in a specific range.. now T is 3 and we know from -0.5 to 0.5 and from 0.5 to 1.5

if I knew what happened from 1.5 to 2.5 then I would have the signal, since it's periodic with fundamental period 3
 
Question: A clock's minute hand has length 4 and its hour hand has length 3. What is the distance between the tips at the moment when it is increasing most rapidly?(Putnam Exam Question) Answer: Making assumption that both the hands moves at constant angular velocities, the answer is ## \sqrt{7} .## But don't you think this assumption is somewhat doubtful and wrong?

Similar threads

  • · Replies 14 ·
Replies
14
Views
2K
Replies
1
Views
2K
  • · Replies 1 ·
Replies
1
Views
2K
  • · Replies 1 ·
Replies
1
Views
2K
  • · Replies 3 ·
Replies
3
Views
2K
  • · Replies 1 ·
Replies
1
Views
1K
  • · Replies 3 ·
Replies
3
Views
2K
  • · Replies 2 ·
Replies
2
Views
2K
  • · Replies 11 ·
Replies
11
Views
2K
  • · Replies 3 ·
Replies
3
Views
2K